1answer.
Ask question
Login Signup
Ask question
All categories
  • English
  • Mathematics
  • Social Studies
  • Business
  • History
  • Health
  • Geography
  • Biology
  • Physics
  • Chemistry
  • Computers and Technology
  • Arts
  • World Languages
  • Spanish
  • French
  • German
  • Advanced Placement (AP)
  • SAT
  • Medicine
  • Law
  • Engineering
mixer [17]
3 years ago
11

Round $7.345 to the nearest cent

Mathematics
1 answer:
Aliun [14]3 years ago
5 0

Answer: $7.35

Step-by-step explanation:

Hi, to round a price to the nearest cent, we have to locate the hundredths place. Then look at the digit to the right. If it is 5 or above, the number in the hundredths place will be increased by 1 and all the rest of the numbers after it are eliminated. if not, the number in the hundredths place remains the same and the rest of the numbers after it are eliminated.

$7.345

The digit in the hundredths place is 4. Since the number after it is 5 we have to increase 4 by 1 and eliminate 5.

$7.35

Feel free to ask for more if needed or if you did not understand something.

You might be interested in
Hello, Brainly community!
ioda

Answer:

(B)  \displaystyle \frac{W(3.1) - W(2.9)}{0.2}

General Formulas and Concepts:

<u>Calculus</u>

Limits

Derivatives

  • The definition of a derivative is the slope of the tangent line.

Derivative Notation

Instantaneous Rates

  • Tangent Line: \displaystyle f'(x) = \frac{f(b) - f(a)}{b - a}

Step-by-step explanation:

Since we are trying to find a <em>rate</em> at which W(t) changes, we must find the <em>derivative</em> at <em>t</em> = 3.

We are given 2 close answer choices that would have the same <em>numerical</em> answer but different <em>meanings</em>:

  1. (A)  \displaystyle  \lim_{t \to 3} W(t)
  2. (B)  \displaystyle \frac{W(3.1) - W(2.9)}{0.2}

If we look at answer choice (A), we see that our units would simply just be volume. It would not have the units of a rate of change. Yes, it may be the closest numerically correct answer, but it does not tell us the <em>rate</em> at which the volume would be changing and it is not a derivative.

If we look at answer choice (B), we see that our units would be cm³/s, and that is most certainly a rate of change. Answer choice (B) is also a <em>derivative</em> at <em>t</em> = 3, and a derivative tells us what <em>rate</em> something is changing.

∴ Answer choice (B) will give us the best estimate for the value of the instantaneous rate of change of W(t) when <em>t</em> = 3.

Topic: AP Calculus AB/BC (Calculus I/I + II)

Unit: Differentiation

Book: College Calculus 10e

8 0
3 years ago
What is the sum of a 56-term arithmetic sequence where the first term is 6 and the last term is 391?
Sati [7]
S_n=\dfrac{a_1+a_n}{2}\cdot n\\\\a_1=6;\ n=56;\ a_{56}=391\\\\subtitute\\\\S_{56}=\dfrac{6+391}{2}\cdot56=\dfrac{397}{2}\cdot56=397\cdot28=11,116
7 0
3 years ago
Read 2 more answers
I need help on these problems
VikaD [51]
Someone is writing the answer so ill just give you this tip for whenever.
A negitive times a negitive is positive
a negative times a positive is negitive
and a positive times a positive is positive (just had to add it to be complete)
8 0
3 years ago
Middle school math with pozzazzi! Book C C-31 what did the football coach send in a bunch of second string players?
Tamiku [17]

Answer:

Step-by-step explanation:

3 0
3 years ago
Carolina is mowing lawns for a summer job. For every mowing job, she charges an initial fee plus $6 sign, for each hour of work.
Shkiper50 [21]

ya the answ is good to hear ok bye t

8 0
3 years ago
Read 2 more answers
Other questions:
  • What operation is being done to the variable in the equation -5m= -40?
    6·1 answer
  • Solve
    10·1 answer
  • State the order of operations in -2(x-5)^2+8:<br><br> 1.<br> 2.<br> 3.
    15·2 answers
  • If you start with a number divide it by 4 then multiply it by two you end up with 8 squared
    14·1 answer
  • Which would be an equivalent way to write y=2−3x?
    14·2 answers
  • two consecutive even integers have a sum of 254. write the equation that will solve for the two integers.
    13·1 answer
  • 141,760.3 minus 66,744.85 equals _______ rounded to the nearest tenth?
    12·2 answers
  • 45 percent of what is 27
    6·1 answer
  • When five months have passed what fraction of the year remains?
    5·2 answers
  • Please answer this question correctly please
    15·1 answer
Add answer
Login
Not registered? Fast signup
Signup
Login Signup
Ask question!